Schwarzschild Circular Orbits Problem

I'm just not sure at all what you're trying to say. Can you please clarify?In summary, the linear velocity of a particle in a circular orbit of radius R in the Schwarzschild geometry, as measured by a stationary observer stationed at one point on the orbit, can be found by using the Schwarzschild metric and the equations derived in chapter 9 of Hartle. By setting the t component of the 4-velocities of the particle and observer equal to each other and using a local inertial coordinate system, the velocity can be determined. However, using the flat space metric in spherical coordinates may not work in this scenario.
  • #1
AuraCrystal
68
0

Homework Statement


Find the linear velocity of a particle in a circular orbit of radius [itex]R[/itex] in the Schwarzschild geometry as would be measured as by a stationary observer stationed at one point on the orbit. (It's problem 10 in chapter 9 of Hartle, if that helps)


Homework Equations


The Scwarzschild metric, obviously,
[itex]ds^2=- \left (1-\frac{2M}{r} \right )dt^2 + \left (1-\frac{2M}{r} \right )^{-1} dr^2 +r^2(d \theta^2 + \sin ^2 \theta d \phi ^2) [/itex]
And in the book they derive the equation (for a circular orbit of radius R)
[itex]\Omega \equiv \frac{d \phi}{d t}= \frac{M}{R^3}[/itex]


The Attempt at a Solution


OK, so in this problem, obviously [itex]\frac{dr}{d \bar{t}}=\frac{d\theta}{d \bar{t}}=0[/itex], where [itex]\bar{t}[/itex] is the time measured in the stationary observer's frame. Ok so we know the angular velocity measured from the far away observer's frame:
[itex]\Omega \equiv \frac{d \phi}{d t}= \frac{M}{R^3}[/itex]
since the stationary observer is, by definition not moving (w.r.t. to our far-away observer), we can easily find [itex]dt/d\bar{t}[/itex] from the metric. We then have [itex]\frac{d \phi}{d \bar{t}}.[/itex] My question is, after we get the velocity 3-vector (i.e. the velocity w.r.t. to the stationary orbit), what do we do and why?
 
Physics news on Phys.org
  • #2
Can you write down the 4-velocity of the particle in orbit and the 4-velocity of the stationary observer?
 
  • #3
George Jones said:
Can you write down the 4-velocity of the particle in orbit and the 4-velocity of the stationary observer?
Of course, it's just:
[tex]u_{observer} = ( dx^{\alpha}_{observer}/d \bar{t} ) [/tex]
[tex]u_{particle} = ( dx^{\alpha}_{particle}/d \tau ) [/tex]
Where α ranges from 0 to 3. (0 is the time component w.r.t. to the observer at ∞).
 
Last edited by a moderator:
  • #4
AuraCrystal said:
Of course, it's just:
[tex]u_{observer} = ( dx^{\alpha}_{observer}/d \bar{t} ) [/tex]
[tex]u_{particle} = ( dx^{\alpha}_{particle}/d \tau ) [/tex]
Where α ranges from 0 to 3. (0 is the time component w.r.t. to the observer at ∞).

Okay, now I have Hartle in front of me. I meant also the specific components of the 4-velocities with respect to Schwarzschild coordinates, i.e., (9.16) for the hovering observer and (9.47) together with (9.48) for the orbiting particle.

What do you get when you divide the t components of the 4-velocities you wrote above? What do you get when you divide the t components of the 4-velocities from the equations I gave?
 
  • #5
^So you want me to write out the components?

[itex]u_{particle}=(\frac{dt}{d \tau} , 0, 0, \frac{d \phi_{particle}}{d \tau}) = ( \sqrt{1-\frac{3m}{R}}, 0, 0, \frac{M}{R^3} )[/itex] (from 9.46, 9.47, and 9.48)
And:
[itex]u_{observer}=(\frac{dt}{d \bar{t}} , 0, 0, 0) [/itex]
From the metric:
[itex]-(d \bar{t})^2=- \left (1-\frac{2M}{r} \right )dt^2 + \left (1-\frac{2M}{r} \right )^{-1} dr^2 +r^2(d \theta^2 + \sin ^2 \theta d \phi ^2) [/itex]
since [itex] (d \bar{t})^2= - ds^2[/itex]. Of course, [itex]d \phi = d \theta = dr = 0 [/itex] since the observer is stationary, so :
[itex](d \bar{t})^2=\left (1-\frac{2M}{r} \right )dt^2[/itex]
Hence, rearranging,
[itex] \left (\frac{dt}{d \bar{t}} \right ) ^2 = \left (1-\frac{2M}{r} \right ) ^ {-1} [/itex]
i.e.,
[itex]\frac{dt}{d \bar{t}} = \left (1-\frac{2M}{r} \right ) ^ {-1/2} [/itex]
so that
[itex]
u_{observer}=(\frac{dt}{d \bar{t}} , 0, 0, 0) = \left ( \left (1-\frac{2M}{r} \right ) ^ {-1/2}, 0, 0, 0 \right ) [/itex]
 
  • #6
AuraCrystal said:
[itex]u_{particle}=(\frac{dt}{d \tau} , 0, 0, \frac{d \phi_{particle}}{d \tau}) = ( \sqrt{1-\frac{3m}{R}}, 0, 0, \frac{M}{R^3} )[/itex] (from 9.46, 9.47, and 9.48)

I am again without Hartle, but shouldn't u^t above be [itex]\left( 1-3m/R \right)^{-1/2}[/itex]?

What is [itex]d \bar{t} / d \tau[/itex] in Schwarzschild coordinates? What is [itex]d \bar{t} / d \tau[/itex] in special relativity?
 
Last edited:
  • #7
^Probably. Typo.

Also:
[itex]d \bar{t} / d \tau = \frac{dt}{d \tau} \frac{d \bar{t}}{dt} = \left ( 1 - \frac{3M}{R} \right ) ^ {-1/2} \left (1-\frac{2M}{R} \right ) ^ {1/2} [/itex]

And in SR, isn't it just the gamma-factor?
 
  • #8
AuraCrystal said:
^Probably. Typo.

Also:
[itex]d \bar{t} / d \tau = \frac{dt}{d \tau} \frac{d \bar{t}}{dt} = \left ( 1 - \frac{3M}{R} \right ) ^ {-1/2} \left (1-\frac{2M}{R} \right ) ^ {1/2} [/itex]

And in SR, isn't it just the gamma-factor?

Yes. What happens when set these expressions equal?
 
  • #9
^So you just solve for v?

Why can you do that? Is it b/c the observer at R is in a local inertial frame?

And if so, can't you just use [itex]v^2 = \eta_{ij} v^{i} v^{j} [/itex] where [itex]\eta_{ij}[/itex] is the flat space metric (in spherical coordinates) and you sum over 1 to 3 and [itex]v^{i}=\left(\frac{dr}{d \bar{t}}, \frac{d \theta}{d \bar{t}}, \frac{d \phi}{d \bar{t}}\right)[/itex]?

...Am I being an idiot here? xD
 
Last edited by a moderator:
  • #10
AuraCrystal said:
^So you just solve for v?

Why can you do that? Is it b/c the observer at R is in a local inertial frame?

Which observer at R? Both observers have constant [itex]r = R[/itex]. The hovering observer H has non-zero 4-acceleration and the orbiting observer O has zero 4-acceleration, so O is inertial and H is not. But, as you suspect, a local inertial coordinate (LIC) can be used to demonstrate the result.

Let [itex]p[/itex] be an event of coincidence for O and H. Let [itex]\left\{ x^{\mu '} \right\}[/itex] be an LIC that has the following properties:

1) [itex]p[/itex] is at the origin;

2) H is at rest at [itex]p[/itex];

3) at [itex]p[/itex], O moves in the positive [itex]x^{1 '}[/itex] direction.

More about 2). Even though H has non-zero 4-acceleration, 2) is possible, but, because of the non-zero 4-acceleration, H will not be at rest away from [itex]p[/itex]. I used H instead of O for 2), because the final is not dependent on one of the observers having zero 4-acceleration.

Let [itex]\mathbf{u}[/itex] and [itex]\mathbf{w}[/itex] be the 4-velocities of H and O respectively. In the LIC, [itex]\mathbf{u}_p = \left( 1, 0, 0, 0 \right)[/itex] and [itex]\mathbf{w}_p = \left( \gamma, \gamma v, 0, 0 \right)[/itex]. Consequently, in the LIC,
[tex]
\gamma = - g_{\mu \nu} u_p^\mu w_p^\nu .
[/tex]
Because [itex]g_{\mu \nu} u_p^\mu w_p^\nu[/itex] is a coordinate-invariant scalar, [itex]\gamma = - g_{\mu \nu} u_p^\mu w_p^\nu[/itex] in any coordinate system in both special and general relativity! This useful result gives the physical (not coordinate) between any two coincident observers. Use Schwarzschild spherical coordinates and this expression to do the problem. In coordinate-free notation,
[tex]\gamma = - g \left( \mathbf{u}_p , \mathbf{w}_p \right)[/tex]
AuraCrystal said:
And if so, can't you just use [itex]v^2 = \eta_{ij} v^{i} v^{j} [/itex] where [itex]\eta_{ij}[/itex] is the flat space metric (in spherical coordinates) and you sum over 1 to 3 and [itex]v^{i}=\left(\frac{dr}{d \bar{t}}, \frac{d \theta}{d \bar{t}}, \frac{d \phi}{d \bar{t}}\right)[/itex]?

This works in Minkowski spacetime, but I don't see how to use this here, and it doesn't work for general curvilinear coordinates in Minkowski spacetime. In Hartle's problem, we can constuct a LIC around [itex]p[/itex], and then use this LIC to construct spherical coordinates locally around [itex]p[/itex], but there won't be an obvious relationship between the local spherical coordinates and Schwarzschild spherical coordinates.
AuraCrystal said:
Am I being an idiot here?
No.
 

Related to Schwarzschild Circular Orbits Problem

1. What is the Schwarzschild Circular Orbits Problem?

The Schwarzschild Circular Orbits Problem is a mathematical problem in the field of general relativity that involves finding the equations of motion for an object moving under the influence of the gravitational field of a non-rotating, spherically symmetric mass, such as a black hole.

2. What are the key concepts involved in the Schwarzschild Circular Orbits Problem?

The key concepts involved in the Schwarzschild Circular Orbits Problem include the Schwarzschild metric, which describes the geometry of space-time surrounding a non-rotating mass, and the geodesic equation, which describes the path an object follows in this curved space-time.

3. How is the Schwarzschild Circular Orbits Problem solved?

The Schwarzschild Circular Orbits Problem is typically solved using the geodesic equation, which involves solving a set of differential equations to determine the equations of motion for an object in the Schwarzschild space-time. This can be done analytically or numerically using computer simulations.

4. What is the significance of the Schwarzschild Circular Orbits Problem?

The Schwarzschild Circular Orbits Problem is significant because it provides a way to study the effects of strong gravitational fields, such as those near black holes. It also allows us to make predictions about the behavior of objects orbiting a massive body, which has practical applications in fields such as astrophysics and space exploration.

5. Are there any real-world applications of the Schwarzschild Circular Orbits Problem?

Yes, there are several real-world applications of the Schwarzschild Circular Orbits Problem. For example, it is used in celestial mechanics to study the motion of planets and satellites around massive bodies. It also has applications in GPS technology, where it is used to calculate the effects of Earth's curvature on satellite orbits.

Similar threads

  • Advanced Physics Homework Help
Replies
10
Views
2K
  • Advanced Physics Homework Help
Replies
5
Views
1K
  • Advanced Physics Homework Help
Replies
10
Views
1K
  • Advanced Physics Homework Help
Replies
4
Views
1K
  • Special and General Relativity
Replies
5
Views
1K
  • Special and General Relativity
Replies
9
Views
1K
  • Advanced Physics Homework Help
Replies
20
Views
2K
  • Advanced Physics Homework Help
Replies
4
Views
2K
  • Advanced Physics Homework Help
Replies
1
Views
1K
  • Advanced Physics Homework Help
Replies
3
Views
1K
Back
Top